User avatar
 
noah
Thanks Received: 1192
Atticus Finch
Atticus Finch
 
Posts: 1541
Joined: February 11th, 2009
 
This post thanked 2 times.
 
 

Q19 - Fares on the city-run

by noah Mon Jun 07, 2010 6:58 pm

The conclusion of this argument is that the town councilors feel that bus fares should be raised. Why? Because these councilors feel that taxes should be used to benefit tax-payers, and some of the folks who benefit from the low fares are commuters (i.e. non-tax payers).

We're asked to find the answer choice that does NOT weaken the argument. Let's knock down the ones that do:

(A) weakens the argument by presenting a reason that raising the fares would HURT the tax-payers, not benefit them.
(B) provides a reason that the higher fares would increase costs.
(C) is strange because it sets up two new premises. Along with establishing that poor folks would be disadvantaged by the higher fares, we learn that the councilors feel that poor folks should be able to take advantage of city-services. With that new rule in place, removing the subsidy doesn't make sense. (C) provides a rather separate reason that the subsidies are problematic.
(D) is out of scope -- there's no mention of concern with what the voters feel.
(E) disrupts the argument by showing that commuters can pay taxes, thus the use of taxes to benefit them would make sense.
 
interestedintacos
Thanks Received: 58
Atticus Finch
Atticus Finch
 
Posts: 116
Joined: November 09th, 2010
 
 
trophy
Most Thanked
trophy
First Responder
 

Re: Q19 - Fares on the city-run

by interestedintacos Mon Feb 28, 2011 2:29 am

I got this question right both times I faced it. But the second time I was especially bothered by answer choice C. Is this a rare type of answer choice for a weaken question?

Can you better explain in precisely what way this choice weakens the argument?

Does its strength come from the fact that the city councillors are advancing the argument?

After all, if we didn't learn that the city councillors cared about the low income non-taxpayers, this answer choice would no longer be a weakener, correct? But because the people making the argument also care about this issue it becomes a weakener?

I don't know; something just seems weird or not right to me. I didn't realize this is a sort of answer on the LSAT:

Tom argues that taco shops should be opened in Springfield because he likes tacos.

Potential weakener: Opening new taco shops in Springfield will attract outsiders to the city Tom believes should be kept out.

Original stimulus:

Some city councillors argue that bus fares should be raised enough to cover the costs of the service

Weakener: Raising bus fees will disadvantage low income residents, and all city councillors agree these residents should be able to use the bus.

Please explain to me how these aren't analogues or if they are analogues...does this pop up on other weaken questions?
 
interestedintacos
Thanks Received: 58
Atticus Finch
Atticus Finch
 
Posts: 116
Joined: November 09th, 2010
 
This post thanked 2 times.
 
trophy
Most Thanked
trophy
First Responder
 

Re: Q19 - Fares on the city-run public buses in

by interestedintacos Mon Feb 28, 2011 2:46 am

Having looked at again I think I came up with something:

The city councillors argue that the city taxes should be used PRIMARILY to benefit the people who pay them.

That means that answer choice C doesn't necessarily conflict with their justification for the conclusion.

In the other answer choices we see cases where people who do pay taxes (and therefore should get benefits) actually lose benefits from the bus fare increase instead of gaining them, as the city councillors intended. So the intention of the councillors is turned around and these weaken their argument.

In answer choice C we are talking about a group we learn is contained within the "other" category implied when we hear that taxpayers are the "primary" category. We essentially learn that this other category is also intended to get benefits, and like the taxpayers in the other choices, instead of gaining benefits they lose them.

By the way, E is the same as the other answer choices, weakening by showing that taxpayers will lose benefits instead of gaining as a result of the fare increase; it's just special because it turns the assumption the argument is based on, that commuters are not taxpayers, on its head.
 
interestedintacos
Thanks Received: 58
Atticus Finch
Atticus Finch
 
Posts: 116
Joined: November 09th, 2010
 
 
trophy
Most Thanked
trophy
First Responder
 

Re: Q19 - Fares on the city-run public buses in

by interestedintacos Mon Feb 28, 2011 3:01 am

Here's another analogy attempt now:

Argument: John argues that taco shops should be the primary businesses to benefit from this year's special tax refund for businesses in Los Angeles, so rules should be passed guaranteeing that Mexican-owned restaurants will get almost all of the refund money available.

Weakeners:
1. Almost all Mexican-owned restaurants in LA are not taco shops.

2 (like C); Mexican-owned grocery stores in LA will have to close if they don't get a significant amount of the refund money, and John believes they should remain open.

So the key point is that 2 doesn't conflict with John's arguing that taco shops should get the prime benefit, just like C doesn't actually conflict with the city councillor's arguing that the prime benefit should go to taxpayers. My mistake was thinking it did--that a weakener could conflict with a premise leaving us to wonder which consideration was more important.
User avatar
 
noah
Thanks Received: 1192
Atticus Finch
Atticus Finch
 
Posts: 1541
Joined: February 11th, 2009
 
 
 

Re: Q19 - Fares on the city-run public buses in

by noah Mon Feb 28, 2011 11:00 am

Interestedintacos, that was awesome. You dug deep into that. I agree, (C) is odd, since it introduces a new premise - but as you mention, it doesn't contradict the others, just introduces a new consideration that disrupts the assured connection between the existing premise and the conclusion.
 
interestedintacos
Thanks Received: 58
Atticus Finch
Atticus Finch
 
Posts: 116
Joined: November 09th, 2010
 
 
trophy
Most Thanked
trophy
First Responder
 

Re: Q19 - Fares on the city-run public buses in

by interestedintacos Mon Feb 28, 2011 9:54 pm

Technically there are a million new things the test makers could do to throw test takers off in logical reasoning. Why not insert a weakener that would create a paradox; then you could argue creating a paradox weakens the argument. As long as the test makers can point out there's only one correct answer, it seems like they could do all kinds of weird things.

Luckily they don't appear to do that, so I'm trying to keep track of the different things they do but still make note of anything special.

By the way, there is a question I found where the premises of an argument conflict. The stimulus simply contains an argument with conflicting premises, and the arguer then concludes by arbitrarily choosing one premise over another, and the test taker is asked to identify this as the flaw of the argument.
 
farhadshekib
Thanks Received: 45
Elle Woods
Elle Woods
 
Posts: 99
Joined: May 05th, 2011
 
 
trophy
Most Thanked
 

Re: Q19 - Fares on the city-run public buses in

by farhadshekib Thu Jun 09, 2011 12:47 pm

interestedintacos Wrote:Having looked at again I think I came up with something:

The city councillors argue that the city taxes should be used PRIMARILY to benefit the people who pay them.

That means that answer choice C doesn't necessarily conflict with their justification for the conclusion.

In the other answer choices we see cases where people who do pay taxes (and therefore should get benefits) actually lose benefits from the bus fare increase instead of gaining them, as the city councillors intended. So the intention of the councillors is turned around and these weaken their argument.

In answer choice C we are talking about a group we learn is contained within the "other" category implied when we hear that taxpayers are the "primary" category. We essentially learn that this other category is also intended to get benefits, and like the taxpayers in the other choices, instead of gaining benefits they lose them.

By the way, E is the same as the other answer choices, weakening by showing that taxpayers will lose benefits instead of gaining as a result of the fare increase; it's just special because it turns the assumption the argument is based on, that commuters are not taxpayers, on its head.



Great post; you really cleared that up for me.

One more thing to note:

the stim says that "SOME city councillors argue that city taxes should be used PRIMARILY to benefit the people who pay them, and therefore that bus fares should be raised enough to cover the cost of the service".

C, however, states that "ALL city councillors agree that these residents [the ones exempt from city taxes] should be able to take advantage of city run services".

So, if "all" city councillors feel this way, it follows that "some" city councillors - those mentioned in the stimulus - must also feel this way.

This is because "all", logically speaking, includes "some".

Therefore, C becomes a weakener because it provides another reason why the bus fares should not be raised.

Let me know if you agree, or if I went wrong in my analysis. Thanks.
User avatar
 
noah
Thanks Received: 1192
Atticus Finch
Atticus Finch
 
Posts: 1541
Joined: February 11th, 2009
 
 
 

Re: Q19 - Fares on the city-run public buses in

by noah Thu Jun 09, 2011 4:12 pm

I agree, the "all" allows us to apply this idea to the "some" councilors - the issue, and the one that interestedintacos delved into quite nicely, is how their belief (and the new fact about poor people) affects this argument.
 
anjelica.grace
Thanks Received: 5
Jackie Chiles
Jackie Chiles
 
Posts: 41
Joined: November 17th, 2011
 
 
 

Re: Q19 - Fares on the city-run public buses in

by anjelica.grace Wed Apr 25, 2012 2:35 am

Sorry. Can someone elaborate how or why concern for voters are out of scope. If they benefit from the lower transit fares and presumably they are taxpayers, why wouldn't they be relevant?
User avatar
 
noah
Thanks Received: 1192
Atticus Finch
Atticus Finch
 
Posts: 1541
Joined: February 11th, 2009
 
 
 

Re: Q19 - Fares on the city-run public buses in

by noah Wed Apr 25, 2012 11:43 am

anjelica.grace Wrote:Sorry. Can someone elaborate how or why concern for voters are out of scope. If they benefit from the lower transit fares and presumably they are taxpayers, why wouldn't they be relevant?

No sorry about it! That's a good question.

Concern for what the voters think would be in scope if the argument core were something like this: council members rely on keeping voters happy --> bus fares should be raised (to end subsidy)

But, instead, the core is this: taxes should be used to benefit the tax-payers --> bus fares should be raised (to end subsidy)

On Assumption Family questions, we need to address the specific argument made. The relationship between the given premise and the conclusion. (D) addresses a different--though reasonable--argument that could be made about the issue.

Does that make sense? This is an important point, so one more example:

Dogs can be violent --> Dogs should be leashed.

What would weaken this?

(D) Dogs know to not pee on flowers.

Yes, this is a good weakener, but of a different argument!

Get it?
 
anjelica.grace
Thanks Received: 5
Jackie Chiles
Jackie Chiles
 
Posts: 41
Joined: November 17th, 2011
 
 
 

Re: Q19 - Fares on the city-run public buses in

by anjelica.grace Wed Apr 25, 2012 5:34 pm

Yes! Understood! Same conclusion but different basis, and thus a different argument, as you said.

Thank you for clearing that up!
 
redcobra21
Thanks Received: 4
Elle Woods
Elle Woods
 
Posts: 59
Joined: July 16th, 2013
 
 
 

Re: Q19 - Fares on the city-run

by redcobra21 Thu Aug 08, 2013 9:44 pm

Hey Noah,

If answer choice (D) had said the same thing but instead of "strongly opposed to increasing local taxes" it was "strongly opposed to increasing bus fares," would that have been a weakener? I get that opposition to a tax hike isn't really relevant to the core of the argument since the conclusion is about increasing fares, but if the taxpayers were opposed to a fare hike, would it be reasonable to assume that they would not benefit from such a course of action? (even though that answer choice would have to do with what voters think)
User avatar
 
noah
Thanks Received: 1192
Atticus Finch
Atticus Finch
 
Posts: 1541
Joined: February 11th, 2009
 
This post thanked 1 time.
 
 

Re: Q19 - Fares on the city-run

by noah Mon Aug 12, 2013 3:22 pm

redcobra21 Wrote:Hey Noah,

If answer choice (D) had said the same thing but instead of "strongly opposed to increasing local taxes" it was "strongly opposed to increasing bus fares," would that have been a weakener? I get that opposition to a tax hike isn't really relevant to the core of the argument since the conclusion is about increasing fares, but if the taxpayers were opposed to a fare hike, would it be reasonable to assume that they would not benefit from such a course of action? (even though that answer choice would have to do with what voters think)

No, it still wouldn't weaken very well since the argument is the connection between the premise about the beneficiaries and the conclusion about raising the fare. As you mention, what does the voter's thoughts have to do with what the council will do?

Most ever LSAT weakener and strengthener is addressing the connection, not simply throwing in a new premise.
 
sportsfan8491
Thanks Received: 12
Forum Guests
 
Posts: 22
Joined: August 28th, 2013
 
 
 

Re: Q19 - Fares on the city-run

by sportsfan8491 Mon Nov 04, 2013 12:35 am

Noah, do you think (D) might actually strengthen the argument somewhat?

I know you said that answer (D) is out of scope, which I sort of agree with after reading your (and all of the other posters') excellent contributions for this question, but I still see answer choice (D) as addressing a subtle assumption that the argument makes even after reading all of the previous posts. When I was doing this in timed practice, I chose (D) because I saw it as somewhat strengthening the councillors' argument by ruling out another possible alternative: ruling out the possibility of actually increasing taxes.

I think an assumption the councillors' argument makes in going from their premise to their conclusion is that raising the taxes to cover the city-run bus service would not be viable solution; hence, the strong conclusion in favor of increasing the bus fares. I know they are talking about 'uses' in their premise, but why wouldn't suggesting an increase in taxes be a viable option? So, in my opinion, this is a consideration that the councillors completely overlook, as we're not told that a tax hike would be completely off the table. That is, a tax increase would allow them to cover the cost of the service as well. It seems like the author is trying to make us overlook this fact by getting us to make a common sense, every-day assumption that tax hikes are bad.

I think answer choice (D) speaks to this tacit assumption by telling us that increasing taxes might actually be detrimental to the city's voters/tax payers because it might make them 'extremely' angry/upset. Given the adverb "strongly" in this answer choice, which denotes a level of 'extreme' seriousness or commitment to a cause that might not exist without it, I saw answer choice (D) as being somewhat of a "strengthen" type answer choice. This is where I think your "out of scope" point also comes into play in my analysis because the argument is talking about what taxes should be used for and not about what taxpayers believe. Nonetheless, it seemed like if I were to throw in answer choice (D) as another premise right before the councillors' conclusion, the conclusion would become a little stronger because a possible alternative (that of increasing taxes) would no longer be a valid consideration.

Thus, in my opinion answer choice (D) makes it seem as though increasing the bus fares would be a more viable option, now that increasing local taxes would be out of the question. I'm not sure if I made an unreasonable 'leap in logic', so I'd appreciate your feedback on this one.
 
mimimimi
Thanks Received: 0
Forum Guests
 
Posts: 19
Joined: March 23rd, 2013
 
 
 

Re: Q19 - Fares on the city-run

by mimimimi Mon Sep 08, 2014 3:16 pm

The argument core is :

City taxes should be used primarily to benefit the people who pay them

Therefore

Bus fare should be raise.

Since (C) is almost like arguing that city taxes should also benefit people who do not pay taxes (low income earners), is it one of the rare LSAT weakeners that actually attack the premise?

Thanks!
 
af10
Thanks Received: 4
Vinny Gambini
Vinny Gambini
 
Posts: 11
Joined: June 30th, 2014
 
 
 

Re: Q19 - Fares on the city-run

by af10 Wed Feb 25, 2015 2:09 pm

Hey,

I'm no expert so take what I say with a grain of salt but, with all due respect, I don't think you explained why (D) is out of scope well, Noah.

I don't think the fact that there isn't any mention of what the voters concern is is what makes it out of scope. It's the fact that the voters are those "who benefit" from taxes ....which doesn't necessarily include those "who pay them." (so slightly out of scope?)

The argument tells us that among those who benefit from low fares are outsiders. The councillors say it should primarily benefit those who "pay them."

For (D), we are only told that "many" (which can be only outsiders) of those who benefit from it are opposed to increasing local taxes. So, we don't know what type of people are included in this many and thus, out of scope/we just aren't sure. I don't think because voters are mentioned means that it is automatically out of scope.

Here's an example of what I mean: (D) (I THINK) would be fine, if it said "Voters who pay these taxes strongly oppose/hate these damn taxes." I think it'd require the small jump to: Hmmm, there's something that WON'T benefit them if they strongly oppose it. but I think that's an OK jump to make.

For (E), I have a few issues but can see why it weakens better than (D). For E, we don't really know if the outsiders earn above the nationally mandated minimum. I thought it was a veryy borderline answer. Is that something we just say "well, it's a minimum so it's more likely than not that that these people earn more than that." ..?


let me know if I'm wrong and completely missed everything lol!
User avatar
 
ohthatpatrick
Thanks Received: 3807
Atticus Finch
Atticus Finch
 
Posts: 4661
Joined: April 01st, 2011
 
This post thanked 3 times.
 
 

Re: Q19 - Fares on the city-run

by ohthatpatrick Mon Mar 02, 2015 2:14 pm

I agree that (E) requires the extra assumption that a significant proportion of the outside-city workers earn above the minimum. But that’s not too big a leap. Strengthen/Weaken answers are far from perfect and often require us to supply some missing idea to make the answer choice actually work.

In terms of (D), I think ‘voters’ are out of scope, but identifying them as ‘people who benefit from the fares’ makes them relevant again.

However, what the heck does raising taxes have to do with anything?

Is the bus fare a tax?

Nope. So their opinion on raising taxes has nothing to do with the proposal we’re evaluating: raising bus fares.

A previous poster brought up an idea I also had … could we consider ‘raising taxes’ to be an alternative solution to the bus dilemma? Rather than raise bus fares, should we raise taxes so that there would be more city tax revenue?

If so, then this answer would have the effect of Strengthening by going against an alternative plan.

However, the current bus dilemma has nothing to do with a lack of tax revenue. We’re just annoyed that outsiders are getting a subsidized bus ride paid for by city taxpayers. So raising taxes has nothing to do with this issue at all.

So again, (D) just has no effect on the argument because we're never talking about raising taxes.

Let us know if questions/qualms remain.
 
roflcoptersoisoi
Thanks Received: 0
Atticus Finch
Atticus Finch
 
Posts: 165
Joined: April 30th, 2015
 
 
 

Re: Q19 - Fares on the city-run

by roflcoptersoisoi Sun Jul 03, 2016 9:09 pm

P: The benefits of taxes should accrue primarily to those that pay them and some of the people that benefit from them are commuters from outside that come in the city to work.
C: We should increase the price of fares enough to cover the cost of service.

One assumption the councillors make is that: Those that commute from outside Greenville to work pay no taxes.


(A) This states a negative consequence of increasing fares, eliminate.
(B) Same as A, eliminate
(C) same as A and B, eliminate
(D) this is completely out of scope. Increase in taxes? Really?
(E) Contradicts an assumption that the councillors make. If they pay taxes then they are part of the group that should benefit from using them, thus making the conclusion (increasing the price of fares) unnecessary.
 
YiX773
Thanks Received: 0
Vinny Gambini
Vinny Gambini
 
Posts: 18
Joined: September 19th, 2018
 
 
 

Re: Q19 - Fares on the city-run

by YiX773 Wed Apr 01, 2020 3:31 am

I have some new thoughts:

Councilors: Premise: If use taxes, then should be used to benefit the payer.
T -> BP
Now not benefit the payer (instead benefit commuters)
Conclusion: Shouldn't use the taxes (should raise fares).

So the councilors appeal to the contrapositive /BP -> /T

Weakeners should be: it benefits the tax payers. (Fail the sufficient condition /BP)

(A) Low fares now are benefiting the tax payers.You see that make beneficial businesses stay here. Weaken.
(B) Same. Low fares now bring us less pollution and lower cost of maintaining fee. Weaken.
(C) New premise. Now T -> BP and T ->LIRB (low income residents benefit). These residents now are beneficiaries. Fail the sufficient condition /LIRB.
(D) Voters? Are they tax payers? Don't know. If they are, then this is a weakener. But that “If...” is an unwarranted assumption.
(E) So the commuters are tax payers. They are beneficiaries. Weaken.